4
$\begingroup$

Theorem 5.3: A measurable function $f$ belongs to $L$ if and only if $|f| $ belongs to $L$.


Definitions: $L = L(X,\mathbf{X}, \mu)$ of integrable functions consists of all real-valued $\mathbf{X}$-measurable functions $f$ defined on $X$, such that both the positive and negative parts $f^{+}, f^{-}$ of $f$ have finite integrals with respect to $\mu$.

A function $f: X \rightarrow \mathbb{R}$ is $\mathbf{X}$-measurable if $f^{-1}((\alpha, \infty)) $ are measurable sets for all real $\alpha$.

If $f: X \rightarrow \mathbb{R}^{+}$ is $\mathbf{X}$-measurable we define the integral with respect to $\mu$ to be the extended real number $$ \int f \, d \mu = \sup \int \phi \, d \mu ,$$ where supremum is taken over all simple functions $\phi: X \rightarrow \mathbb{R}$ such that $ 0 \le \phi \le f $. (The integral of simple function is then the usual one with $\mu$.)


What I don't see: $|f| \in L \Rightarrow f \in L$

We have $|f|^{+} = f^{+} + f^{-} , |f|^{-} = 0$ have finite integrals. But this does even not imply that $f^{+}$ and $f^{-}$ are $\mathbf{X}$- measurable. What am I missing?


EDIT: As given in the counter example by the comments, I believe the theorem should be reformulated as,

Let $f$ be a measurable function, then $f \in L \Leftrightarrow |f| \in L$

Is this correct?

$\endgroup$

3 Answers 3

2
$\begingroup$

This is not true, an easy counterexample is to let $A$ be a non-measurable subset of $[0,1]$ and define $$f : [0,1] \rightarrow \mathbb{R}, \; \; f(x) = \begin{cases} 1: & x \in A; \\ -1: & x \notin A \end{cases}.$$

$\endgroup$
2
$\begingroup$

The definition of integrable should include that $f$ is measurable. Otherwise, take $1-2\,\chi_E$ for any non-measurable set $E$, and its absolute value will be $1$.

$\endgroup$
2
  • $\begingroup$ Thanks, I edited. Also, it seems like the original theorem should be formulated in the new way I edited - does that seem right? $\endgroup$
    – Bryan Shih
    Dec 26, 2016 at 5:12
  • $\begingroup$ Now I see that you already had measurability as a condition in $L $. The theorem is simply $f\in L\iff |f|\in L $. $\endgroup$ Dec 26, 2016 at 7:20
1
$\begingroup$

$\vert f \vert = f^+ + f^-$. Then if $\vert f \vert$ belongs to $L$ it means that both $f^+$ and $f^-$ have finite integrals because we already know that the integral is linear (and the integral is defined over every measurable function and $f^+,f^-$ are measurable). Hence, if $\int f^+d\mu$ or $\int f^-d\mu$ were infinite, it wouldn't be the case that $\int \vert f \vert d\mu$ is finite.

$\endgroup$
1
  • $\begingroup$ you also can see that $\vert \int f \ d\mu \vert \leq \int f⁺ \ d\mu + \int f^- \ d\mu = \int \vert f \vert \ d\mu$. So if the integral on the right is finite, so is the other on the left. $\endgroup$
    – Marcelo
    Sep 11, 2018 at 22:42

You must log in to answer this question.

Not the answer you're looking for? Browse other questions tagged .